LSAT and Law School Admissions Forum

Get expert LSAT preparation and law school admissions advice from PowerScore Test Preparation.

User avatar
 Dave Killoran
PowerScore Staff
  • PowerScore Staff
  • Posts: 5852
  • Joined: Mar 25, 2011
|
#94656
Complete Question Explanation
(The complete setup for this game can be found here: lsat/viewtopic.php?f=298&t=12774)

The correct answer choice is (C)

The Not Laws produced by the first rule show that J can never be assembled on line 1, and thus the lowest-numbered line that J can be assembled on is line 2. Hence, answer choice (C) is correct.
 rita02
  • Posts: 6
  • Joined: Jun 25, 2020
|
#78299
I am having trouble understanding why C is the correct answer here. I thought it is possible for J to be assembled on line 3. The order of trucks would then be HFJSMGK. I am unsure what I am missing here.
 Paul Marsh
PowerScore Staff
  • PowerScore Staff
  • Posts: 290
  • Joined: Oct 15, 2019
|
#78388
Hi Rita - It's definitely possible for J to be assembled on line 3! It could also be assembled on lines 2, 5, 6, and 7. But the question is asking what the lowest possible assembly line number is for one of our variables. In other words - what's the earliest that variable can go?

We know from Rule 1 that F has to come before J, so J can't be assembled on line 1. That means the lowest-numbered line on which J can be assembled is line 2. So (C) is our right answer.

We can cross out the other answer choices because F, K, and M could all be assembled as low as line 1; G can be assembled as low as line 2.

Hope that helps!
 rita02
  • Posts: 6
  • Joined: Jun 25, 2020
|
#78398
Thank you, that explains it! I misunderstood the wording in this question.

Get the most out of your LSAT Prep Plus subscription.

Analyze and track your performance with our Testing and Analytics Package.